Last visit was: 23 Apr 2024, 14:51 It is currently 23 Apr 2024, 14:51

Close
GMAT Club Daily Prep
Thank you for using the timer - this advanced tool can estimate your performance and suggest more practice questions. We have subscribed you to Daily Prep Questions via email.

Customized
for You

we will pick new questions that match your level based on your Timer History

Track
Your Progress

every week, we’ll send you an estimated GMAT score based on your performance

Practice
Pays

we will pick new questions that match your level based on your Timer History
Not interested in getting valuable practice questions and articles delivered to your email? No problem, unsubscribe here.
Close
Request Expert Reply
Confirm Cancel
SORT BY:
Kudos
Tags:
Show Tags
Hide Tags
User avatar
Manager
Manager
Joined: 15 Apr 2010
Posts: 83
Own Kudos [?]: 342 [33]
Given Kudos: 3
Send PM
Most Helpful Reply
User avatar
Kaplan GMAT Instructor
Joined: 21 Jun 2010
Posts: 55
Own Kudos [?]: 451 [5]
Given Kudos: 2
Location: Toronto
Send PM
General Discussion
Senior Manager
Senior Manager
Joined: 26 Nov 2009
Status:Impossible is not a fact. It's an opinion. It's a dare. Impossible is nothing.
Affiliations: University of Chicago Booth School of Business
Posts: 470
Own Kudos [?]: 2377 [4]
Given Kudos: 36
Location: Singapore
Concentration: General Management, Finance
Schools: Chicago Booth - Class of 2015
Send PM
User avatar
Kaplan GMAT Instructor
Joined: 21 Jun 2010
Posts: 55
Own Kudos [?]: 451 [2]
Given Kudos: 2
Location: Toronto
Send PM
Re: A certain drug, when taken by patients with high blood pressure, was [#permalink]
2
Kudos
Hi,

great questions guys! Let's start with nusmavrik's:

Quote:
Hi Testluv
Thanks. How do you this is assumption question?


Because the question stem asks for something that the conclusion's validity DEPENDS on. An idea on which a conclusion depends is a necessary assumption. Let me know if you seek further elaboration

Here are dwivedys' questions:

Quote:
If the high bp people both use drugs and exercise frequently and lets say that by doing both they were able to keep their bp in check - to which of the two would we attribute this effect to? (to exercise or to drug?)


That would indeed be a conundrum. Fortunately, we don't have to deal with it. The author's evidence already established that this group of high-blood pressure-patients' reduced their blood pressure by taking the drug. Thus, their blood pressure was high UNTIL they took the drug.

Now, if we learn that (many of) the high-blood-pressure patients were also regular exercisers (as per the denial of choice D), then we know that this group of patients had high blood pressure even though they are regular exercisers. Basically, we'd have a group of people who exercised regularly and who had high blood pressure who then alleviated their high blood pressure by taking the drug. That seriously damages the author's argument that the drug is not as effective as regular exercise.

Quote:
what do we mean by saying Exercise is more effective in keeping bp under check?


As always, we have to attach a commonsense interpretation given the context of the argument. Look at the first sentence: "a certain drug...was found to lower blood pressure". Thus, the only thing the author could possibly intend to mean when he claims that the drug is "not as effective" as exercise is that exercise is better at lowering blood pressure.

Quote:
I am not convinced its a good quality question


I don't see any design flaws with this question!
User avatar
Manager
Manager
Joined: 24 Dec 2009
Posts: 116
Own Kudos [?]: 124 [1]
Given Kudos: 3
Send PM
Re: A certain drug, when taken by patients with high blood pressure, was [#permalink]
1
Kudos
Answer should be D. But can some one please explain the solution by logically negating the answer choice. I believe the logical negation of the answer choice D will be-

None of the patients with high blood pressure exercise regularly. I am not sure if this statement is weakening the argument. Thank You.

Thanks,
Akhil M.Parekh
User avatar
Current Student
Joined: 29 Apr 2010
Posts: 196
Own Kudos [?]: 48 [1]
Given Kudos: 26
Concentration: Cleantech
Schools:Sloan R1, McCombs R1, Ross R1 (w/int), Haas R2, Kellogg R2
WE 1: Product Engineering/Manufacturing
Send PM
Re: A certain drug, when taken by patients with high blood pressure, was [#permalink]
1
Kudos
nusmavrik wrote:
OA is wrong. I fact i'm sure about it.
However A fixes the argument.
The argument doesn;t talk about the low blood pressure. Choice A means that regardless of the blood pressure, patients can benefit from the exercise.Hence drug is not as effective as the exercise.

D limits the scope of the argument making it untenable.


A is too strong to be a correct GMAT answer, IMO.
Senior Manager
Senior Manager
Joined: 26 Nov 2009
Status:Impossible is not a fact. It's an opinion. It's a dare. Impossible is nothing.
Affiliations: University of Chicago Booth School of Business
Posts: 470
Own Kudos [?]: 2377 [1]
Given Kudos: 36
Location: Singapore
Concentration: General Management, Finance
Schools: Chicago Booth - Class of 2015
Send PM
Re: A certain drug, when taken by patients with high blood pressure, was [#permalink]
1
Bookmarks
Hi Testluv
Thanks. How do you this is assumption question? Sorry I think I missed the type of Q. Can you please elaborate?

Here are a few examples of questions that signal an assumption question:
Which of the following is an assumption on which this argument depends?
The explanation above assumes that......
Which of the following is an assumption made in drawing the conclusion above?
The conclusion drawn above is based on the assumption that

Testluv wrote:
As another poster pointed out, choice A is extreme for an assumption question.
Director
Director
Joined: 03 Feb 2013
Posts: 797
Own Kudos [?]: 2588 [1]
Given Kudos: 567
Location: India
Concentration: Operations, Strategy
GMAT 1: 760 Q49 V44
GPA: 3.88
WE:Engineering (Computer Software)
Send PM
Re: A certain drug, when taken by patients with high blood pressure, was [#permalink]
1
Bookmarks
A certain drug, when taken by patients with high blood pressure, was found to lower blood pressure to high-normal levels, or from an average of 30 percent to an average of 10 percent above normal. However, a survey of patients with normal blood pressure found that almost 80 percent exercised at least 5 days per week for 30 minutes and maintained a normal blood pressure. Therefore, the drug is not as effective in reducing blood pressure in patients with high blood pressure as is exercise.
The validity of the above conclusion depends on the truth of which of the following?

Premise 1 : Drug decreased from 30% to 10% above normal.
Premise 2 : 80% exercised frequently and could maintain normal blood pressure.
Conclusion : Exercising is more effective than drug

A. Frequent exercise has the same effect on blood pressure in all patients.
Same effect -> high blood pressure patients can maintain the blood pressure. I presume high remains high - It doesn't make sense and refutes the author's argument.

B. No patient with high blood pressure successfully achieved below-normal levels with the drug alone.
Part of the premise : The patients decreased 30% to 10% - Cannot be the assumption.

C. Patients with normal blood pressure maintained healthy diets for the duration of the study.
- Out of scope

D. Few of the patients with high blood pressure exercise regularly.
- Negation : Most of the patients with high blood pressure exercise regularly - Breaks the conclusion

E. The drug has little or no effect on patients with normal blood pressure.
- Out of scope.

Hence D)
User avatar
Manager
Manager
Joined: 16 Jun 2010
Posts: 69
Own Kudos [?]: 103 [0]
Given Kudos: 1
 Q48  V36
Send PM
Re: A certain drug, when taken by patients with high blood pressure, was [#permalink]
D . Concidering that sample set of people, who generally excercise, consists of some peole who have high blood presure.
User avatar
Director
Director
Joined: 15 Jul 2004
Posts: 603
Own Kudos [?]: 673 [0]
Given Kudos: 17
Concentration: Strategy
Schools:Wharton (R2 - submitted); HBS (R2 - submitted); IIMA (admitted for 1 year PGPX)
 Q48  V33 GMAT 2: 670  Q46  V36 GMAT 3: 720  Q49  V40
Send PM
Re: A certain drug, when taken by patients with high blood pressure, was [#permalink]
in order to conclusively prove that exercise is more effective than the drug in keep bp under check we must create two samples of people - one who exercise and dont take drug and the other who dont exercise but take the drug. Then we should compare their blood pressures. If "normal" people tend to maintain healthy bp with exercise and those who take drug are not able to bring their bp to normal levels - it shows that the drug has failed to be effective.

I chose D purely by elimination because the other choices were making no sense. In fact I believe its a poor quality question because we cannot logically compare healthy people who dont have bp to those who have --- those with BP may or may not respond to exercise in the same way as normal people do (for whom exercise alone may be sufficient to maintain healthy bp).
User avatar
Director
Director
Joined: 15 Jul 2004
Posts: 603
Own Kudos [?]: 673 [0]
Given Kudos: 17
Concentration: Strategy
Schools:Wharton (R2 - submitted); HBS (R2 - submitted); IIMA (admitted for 1 year PGPX)
 Q48  V33 GMAT 2: 670  Q46  V36 GMAT 3: 720  Q49  V40
Send PM
Re: A certain drug, when taken by patients with high blood pressure, was [#permalink]
Testluv wrote:
Hi,

the correct answer is definitely choice D. We can use the Kaplan denial test to prove it. The author concludes that the drug isn't as effective as exercise in treating high blood pressure. Choice D reads:

"FEW of the patients with high blood pressure exercise regularly"

whose denial is:

"MANY of the patients with high blood pressure exercise regularly"

in which case the argument falls apart (because even though they exercise their blood pressure didn't go down until they were on the drug.) So, when D is false, the argument can't stand. Thus, the argument depends on choice D.

As another poster pointed out, choice A is extreme for an assumption question. The author's reasoning does not depend on the idea that exercise has the same effect on blood pressure in ALL patients. For example, does he have to assume exercise will have a similar effect in LOW-blood pressure patients? Of course not. Thus, his reasoning does NOT depend on choice A.

Another poster pointed out that the argument uses flawed reasoning in that it illogically compares two different groups. Although that's true, that certainly doesn't mean that it is not a good question. In many arguments, the arguer's reasoning will be flawed.


If the high bp people both use drugs and exercise frequently and lets say that by doing both they were able to keep their bp in check - to which of the two would we attribute this effect to? (to exercise or to drug?) it would still leave open the possibility that it was NOT the exercise but the drug that was keeping the bp normal and hence comparing this set of people with the set that exercises regularly and keeps bp in check would not yield a conclusive result.

Besides - what do we mean by saying Exercise is more effective in keeping bp under check? For normal people BP is not an issue to begin with so how can we say that exercise is helping them keep the bp in check? If they had BP to begin with and then they exercised and kept BP in check then we could conclude that exercise is perhaps more effective than the drug is in keeping the BP in check. I am not convinced its a good quality question but anyway... it's not a big deal.
User avatar
Director
Director
Joined: 15 Jul 2004
Posts: 603
Own Kudos [?]: 673 [0]
Given Kudos: 17
Concentration: Strategy
Schools:Wharton (R2 - submitted); HBS (R2 - submitted); IIMA (admitted for 1 year PGPX)
 Q48  V33 GMAT 2: 670  Q46  V36 GMAT 3: 720  Q49  V40
Send PM
Re: A certain drug, when taken by patients with high blood pressure, was [#permalink]
Thanks...

Your explanation does make sense. It just didn't occur to me that way when I read the question first.

Testluv wrote:
Hi,

great questions guys! Let's start with nusmavrik's:

Quote:
Hi Testluv
Thanks. How do you this is assumption question?


Because the question stem asks for something that the conclusion's validity DEPENDS on. An idea on which a conclusion depends is a necessary assumption. Let me know if you seek further elaboration

Here are dwivedys' questions:

Quote:
If the high bp people both use drugs and exercise frequently and lets say that by doing both they were able to keep their bp in check - to which of the two would we attribute this effect to? (to exercise or to drug?)


That would indeed be a conundrum. Fortunately, we don't have to deal with it. The author's evidence already established that this group of high-blood pressure-patients' reduced their blood pressure by taking the drug. Thus, their blood pressure was high UNTIL they took the drug.

Now, if we learn that (many of) the high-blood-pressure patients were also regular exercisers (as per the denial of choice D), then we know that this group of patients had high blood pressure even though they are regular exercisers. Basically, we'd have a group of people who exercised regularly and who had high blood pressure who then alleviated their high blood pressure by taking the drug. That seriously damages the author's argument that the drug is not as effective as regular exercise.

Quote:
what do we mean by saying Exercise is more effective in keeping bp under check?


As always, we have to attach a commonsense interpretation given the context of the argument. Look at the first sentence: "a certain drug...was found to lower blood pressure". Thus, the only thing the author could possibly intend to mean when he claims that the drug is "not as effective" as exercise is that exercise is better at lowering blood pressure.

Quote:
I am not convinced its a good quality question


I don't see any design flaws with this question!
GMAT Club Legend
GMAT Club Legend
Joined: 03 Oct 2013
Affiliations: CrackVerbal
Posts: 4946
Own Kudos [?]: 7624 [0]
Given Kudos: 215
Location: India
Send PM
Re: A certain drug, when taken by patients with high blood pressure, was [#permalink]
amp0201 wrote:
Answer should be D. But can some one please explain the solution by logically negating the answer choice. I believe the logical negation of the answer choice D will be-

None of the patients with high blood pressure exercise regularly. I am not sure if this statement is weakening the argument. Thank You.

Thanks,
Akhil M.Parekh



Hey Akhil,
The problem here is more language than anything else.
"Few of the patients with high blood pressure exercise regularly" would mean that NO ONE exercised.
(A few, on the other hand, means that there were a couple or more individuals that did exercise. )
Therefore on negation the statement would read "ALL/ MOST of the patients with high BP exercise regularly"

Hope that helps,
Ajeeth Peo
avatar
Intern
Intern
Joined: 29 Sep 2012
Posts: 7
Own Kudos [?]: 20 [0]
Given Kudos: 1
Send PM
Re: A certain drug, when taken by patients with high blood pressure, was [#permalink]
Hi,

I just gave a test and came across this question. However, the answer choices are slightly different. Here's the question:

A certain drug, when taken by patients with high blood pressures, was found to lower the blood pressure to high-normal levels, or from an average of 30 percent to an average of 10 percent above normal. However a survey of patients with normal blood pressures found that almost 80 percent exercised atleast 5 days per week for 30 minutes and maintained a normal blood pressure.
Therefore, the drug is not as affective in reducing blood pressure in patients with high blood pressure as is exercise. The validity of the above conclusion depends on the truth of which of the following ?

A. None of the patients with high blood pressure exercise regularly.
B. No patient with high blood pressure successfully achieved below-normal levels with drug alone.
C. Patients with normal blood-pressure maintained healthy diets for the duration of the study.
D. Daily exercise has the same effect on the blood pressures in all patients.
E. The drug has little or no effects on patients with normal blood pressures.

The OA for this is D. However, the original post does not support this OA.

During the test, I had narrowed down to A and D, and I've read some convincing explanations for both, however, I'm still confused between A and D.

Experts- Kindly help me eliminate A and justify D.

Thanks & Regards,
Chandni
Director
Director
Joined: 03 Feb 2013
Posts: 797
Own Kudos [?]: 2588 [0]
Given Kudos: 567
Location: India
Concentration: Operations, Strategy
GMAT 1: 760 Q49 V44
GPA: 3.88
WE:Engineering (Computer Software)
Send PM
Re: A certain drug, when taken by patients with high blood pressure, was [#permalink]
Chandni170 wrote:
Hi,

I just gave a test and came across this question. However, the answer choices are slightly different. Here's the question:

A certain drug, when taken by patients with high blood pressures, was found to lower the blood pressure to high-normal levels, or from an average of 30 percent to an average of 10 percent above normal. However a survey of patients with normal blood pressures found that almost 80 percent exercised atleast 5 days per week for 30 minutes and maintained a normal blood pressure.
Therefore, the drug is not as effective in reducing blood pressure in patients with high blood pressure as is exercise. The validity of the above conclusion depends on the truth of which of the following ?

A. None of the patients with high blood pressure exercise regularly.
B. No patient with high blood pressure successfully achieved below-normal levels with drug alone.
C. Patients with normal blood-pressure maintained healthy diets for the duration of the study.
D. Daily exercise has the same effect on the blood pressures in all patients.
E. The drug has little or no effects on patients with normal blood pressures.

The OA for this is D. However, the original post does not support this OA.

During the test, I had narrowed down to A and D, and I've read some convincing explanations for both, however, I'm still confused between A and D.

Experts- Kindly help me eliminate A and justify D.

Thanks & Regards,
Chandni


I am going to discuss only the two options A and D.

Premise 1: Drugs can reduce blood pressure to some extent and not completely.
Premise 2 : 80% of the patients with normal blood pressure can maintain their blood pressure if they exercise.
Conclusion : Exercise is better than Drugs.

A big leap of faith. Drugs doesn't work on high blood pressure and using exercise the patients with normal blood pressure can maintain their blood pressure.

A. None of the patients with high blood pressure exercise regularly.
This statement isolates the effect to only the drugs and not the to exercise. Still we cannot compare the effects and come to a conclusion.

D. Daily exercise has the same effect on the blood pressures in all patients.
Now, this connects the dots. Daily exercise has the same effects on all patients - Including high blood pressure patients.
So high blood pressure patients can have normal blood pressure if they exercise regularly.
Intern
Intern
Joined: 07 Jan 2014
Posts: 1
Own Kudos [?]: [0]
Given Kudos: 5
Send PM
Re: A certain drug, when taken by patients with high blood pressure, was [#permalink]
Hi,

Conclusion: Drug is not as effective as exercise is in reducing blood pressure in patients with high blood pressure.

Premise 1: Certain drug can lower BP in patients with high BP to high-normal levels (from an avg 30% above normal to 10% above normal)
Premise 2: Survey of patients with normal BP found that almost 80% exercised at least 5 days per week for 30 minutes and maintained a normal BP.

Now, based on these premises the author has concluded that drug is not as effective as exercise is in reducing BP in patients.

Lets use the falsification method.

Q. Under what conditions drug will be as effective as exercise is in reducing the blood pressure in patients?

1. The patients that maintained normal blood pressure could do so by exercising as well as taking the drug as with only exercising they are able to maintain their blood pressure levels above 10% of normal.


Assumption (Negate the above statement):

Patients with normal blood pressure levels are able to maintain normal BP only with exercise.

or, we can say that

Patients that take drugs, few if any (means almost none of them) exercise regularly.


Therefore, option D is the correct answer.

- Thanks
User avatar
Non-Human User
Joined: 01 Oct 2013
Posts: 17206
Own Kudos [?]: 848 [0]
Given Kudos: 0
Send PM
Re: A certain drug, when taken by patients with high blood pressure, was [#permalink]
Hello from the GMAT Club VerbalBot!

Thanks to another GMAT Club member, I have just discovered this valuable topic, yet it had no discussion for over a year. I am now bumping it up - doing my job. I think you may find it valuable (esp those replies with Kudos).

Want to see all other topics I dig out? Follow me (click follow button on profile). You will receive a summary of all topics I bump in your profile area as well as via email.
GMAT Club Bot
Re: A certain drug, when taken by patients with high blood pressure, was [#permalink]
Moderators:
GMAT Club Verbal Expert
6917 posts
GMAT Club Verbal Expert
238 posts
CR Forum Moderator
832 posts

Powered by phpBB © phpBB Group | Emoji artwork provided by EmojiOne